Sample Test
Sample Test
Entrance Tests
For
Vegetarian food can be healthier than a traditional diet. Research has shown that
vegetarians are less likely to suffer from heart disease and obesity than meat eaters.
Concern has been expressed that vegetarians do not get enough protein in their diet but it
has been demonstrated that, by selecting foods carefully, vegetarians are able to amply
meet their needs in this respect.
Which of the following best expresses the main conclusion of the above argument?
A balanced diet is more important for health than any particular food.
Solution
What does this argument seem to be trying to get you to accept? It seems to be trying to persuade you that
vegetarian food can be healthier than a traditional diet (the first sentence). If you think this is the main
conclusion, you should then check whether the rest of the passage gives you reason to believe this. Two
reasons are given:
1. Vegetarians are less likely to suffer from heart disease and obesity than meat eaters.
You may not know whether these reasons are true, but if they were true, they would indicate that vegetarian
food is healthier in one respect than a diet which includes meat, and that a vegetarian diet does not necessarily
have the disadvantage to health (providing insufficient protein) which some may think. So it seems clear that
the first sentence of the passage is being offered as a conclusion. A is the correct answer.
Which one of the following conclusions can be drawn from the passage?
As the demand for blood has increased, so has the supply fallen.
The rate of growth of the blood-donor population has been slowing recently.
The increase in the rate of demand for blood is mainly due to population growth.
If more blood donors could be found, there would be no need to find a substitute for
human blood.
The problem of the increase in demand for blood shows no sign of disappearing.
Solution
The answer to this question is E, because the passage makes it clear that demand for blood is growing, and
there is still no substitute available.
A does not follow from the passage, because although it states that supply has not kept pace with demand, it
does not state that the supply has fallen.
B does not follow from the passage, because the passage does not give any specific information about the rate
of growth of the blood donor population, apart from stating that it has not kept pace with demand.
C does not follow from the passage; it does not make any claims about the general rate of population growth.
D does not follow from the passage, because although it may be true, it is not of direct relevance to the
argument.
There are enough teachers for the courses which would be subsidised.
Adults attending these courses will be able to upgrade their jobs if they pass the
examinations.
Solution
This question asks you to identify what must be assumed in order for the conclusion to be valid. Although
options A, B and C may seem plausible, they are of little relevance to the argument. Whilst option D might be
true, it does not have to be assumed, because the argument is based on the study’s evidence of the benefits of
adult education. It is E, courses which are purely for leisure cannot be economically useful, that must be
assumed if the argument is to conclude that such courses should not be funded.
Any work with emotional force and complexity is capable of being great art.
The decision of the jury to award the prize for a video was the right one.
No-one can really answer the question: What is great art?
This year’s winning exhibit was deceptively simple.
Painting and sculpture are the highest forms of creative art.
Solution
B is neither assumed nor stated. The argument would hold even if the jury were wrong, as it has a
conditional conclusion (if they are right in this evaluation…)
C would not be a welcome assumption at all, in an argument which is trying to give a limited definition of what
art is.
D would be implied if the jury was right in its evaluation, but is not assumed.
E Painting and sculpture are cited as examples of accepted art forms, but there is no assumption that they are
the highest forms.
Which one of the following is the best statement of the flaw in the argument above?
A Critics are often wrong in their predictions about the popularity of films.
B The cost of making a film is normally greater than its original budget.
C The cost of a film need not be the factor that determines its popularity.
D The popularity of a film would justify a high level of expense in making it.
E The public does not necessarily know whether a film is expensive or cheap to make.
Solution
The answer is C. The argument draws the conclusion that if directors want to make popular films, they should
stick to low budgets. The reasoning offered in support of this is that:
(i) some recent films have been very expensive to make, but have not been successful enough to justify the
expense;
(ii) there have been films made very cheaply that have been very popular; and
(iii) some directors who have made successful low-budget films have gone on to make unsuccessful but
expensive films.
But the conclusion does not follow, because the argument fails to establish a causal link between the cost of
making a film and its popularity; it fails to consider high budget films that have been popular and low budget
films that have been unpopular. C is the statement which best explains this.
A does not describe the flaw, because it simply states something that may be true, but is of little relevance to
the argument.
B does not describe the flaw, because the argument does not depend upon a comparison of original and final
budgets.
D does not describe the flaw, but states something that may be true but, if anything, contradicts the conclusion
of the argument.
E does not describe the flaw, because it simply states something that may be true but is not relied upon by the
argument when reaching the conclusion.
Which one of the following is an expression of the main conclusion in the argument above?
A It is easier to do things in a country if you have learnt the language of that country.
B You will find out more about a country if you learn the language of that country.
D When people visit a foreign country, they should try to learn some of the language.
Solution
The argument starts with the recommendation that if people go to a foreign country, they should try to learn
at least some of the language of the country. This is followed by three reasons for acting upon this
recommendation. They are:
• Learning a little of a foreign language helps you to find out more about the country itself and its
people’s customs.
• You can do things (for example, asking for directions or ordering a meal) much more easily.
• Using the language of the country is less embarrassing than pointing and arm-waving.
Thus the conclusion of the argument is the recommendation with which it starts, and this is best expressed
in D.
A is the second reason for the conclusion. B is the first reason for the conclusion. C points out a possible
objection to the argument, which is then disregarded. E is a combination of the second and third reasons.
Which one of the following, if true, would most weaken the above argument?
B Schoolchildren can learn an awful lot about animals from visiting zoos.
C Many of the animals kept in zoos would not be capable of surviving in the wild.
E Zoos enable endangered species to survive by breeding them in captivity and then
re-introducing them to the wild.
Solution
The argument recommends closing down zoos and reallocating the money saved to the protection of natural
habitats on the grounds that zoos are unsuitable places for animals, because the crowded and unnatural
conditions cause animals to behave in abnormal and neurotic ways. If E is true, then zoos have a valuable
function in relation to endangered species, enabling them to breed safely in captivity then be re-introduced to
the wild, where they will be able to live in their natural environment. So E weakens the argument that all zoos
should be closed down because of their unsuitability for animals.
A does not weaken the argument because the fact that living in crowded conditions can make humans
neurotic is not a good reason for keeping animals in unnatural and crowded conditions in zoos.
B does not provide a strong reason for retaining zoos, since children who visit zoos may be learning what
different animals look like (which they could also learn from books and television), but they are not learning
about how animals behave in their natural environment. So B does not substantially weaken the argument.
C provides a reason for managing closure of zoos in such a way that those animals which would be unable
survive would not be released into the wild. But it does not weaken the argument because it does not
provide a good reason for not closing down zoos eventually.
Assuming that it is a good thing to protect habitats, D is a good reason for seeking new sources of funding
for habitat protection. However, it neither weakens nor strengthens the argument, because the high cost of
habitat protection implies neither that habitat protection is too costly to be worth doing, nor that zoos should
be closed in order to meet the cost.
Which one of the following most closely parallels the reasoning used in the above
argument?
A Amrik does not like foods containing garlic. This pizza contains garlic and
anchovies so Amrik will not like it either.
B It is too far for Amrik to walk to the garden centre. The shops are closer so, he will
go there instead.
C Amrik cannot sleep at night if he drinks a cup of tea after 9pm, because tea
contains caffeine. Coffee contains more caffeine than tea, so Amrik won’t drink that
after 9pm either.
D Amrik didn’t have enough patience to complete the 1000-piece jigsaw he got for his
birthday. The Times crossword also requires patience, so he will not complete that
either.
E Amrik’s hair is shorter, and Callum’s hair is longer, than Bill’s. So Amrik’s hair is
shorter than Callum’s.
Solution
C is the correct answer. The structure of the original passage, and of response C, can be expressed in the
following way:
Amrik cannot X because X is too Y.
Z is more Y.
So Amrik cannot Z.
In the original passage X = buy(ing) the Advanced version, Y = expensive, Z = buy(ing) the Professional
version.
In response C, X = drink(ing) tea after 9pm, Y = contains caffeine, Z = drink(ing) coffee after 9pm.
In A, the structure is: Amrik does not like foods with X (garlic). Y (pizza) contains X, so Amrik does not like
Y.
In B, the structure is: Amrik cannot X (walk to the garden centre) because X is too Y (far), Z (walking to the
shops) is not too Y, so Amrik will do Z.
In D, the structure is: Amrik had too little X (patience) to do Y (a jigsaw). Z (doing a crossword) requires X,
so Amrik will not do Z.
In E, the structure is: X (Amrik’s hair) is shorter than Y (Bill’s hair) and Y is shorter than Z Callum’s hair), so
X is shorter than Z.
Which one of the following is the best expression of the flaw in the above argument?
B It assumes that there is a single part of the brain that is responsible for one’s IQ.
D It does not state how many IQ tests constitute a very large number.
Solution
The argument describes a study which compared the brains of adult humans before and after periods of
intense memory recall. The argument suggests that because the parts of the brain responsible for memory
had increased in size at the end of the task of memorising, the brain must increase in size and power the
more it is used. From this it draws the conclusion that those who want to improve their IQ should take a
large number of IQ tests.
But the study shows only that tasks of memorising can increase the size of the brain, and that, if such tasks
do increase the power of the brain, we can only be certain that it is memory power that is increased. So the
general conclusion that assumes that IQ would be improved by taking a large number of IQ tests is not
supported by the evidence. C is the option which expresses this flaw.
A does not express a flaw because the argument does not say that London taxi drivers were the only
people studied. Taxi drivers are mentioned merely as an example.
B does not express a flaw because the argument does not refer to different parts of the brain.
The argument does not say how many IQ tests constitute a large number, but D does not express a flaw
because this point is not relevant to the way in which the conclusion does not follow from the evidence.
The argument does assume that the increase in brain size is an indication of increase in brain power, but
the flaw in the argument lies in the move from ‘increase in memory power’ to ‘increase in general
intelligence’. E does not identify this flaw.
The following table gives figures for the percentage growth per year of labour productivity
per person per year in various countries during three periods.
Which country's percentage growth per year remained consistently greater than half of its
Period 1 level in the following periods?
A Belgium
B Denmark
C France
D Germany
E United Kingdom
Solution
For this question, you need first to be clear what you need to do to find the answer: you must identify which row
of the table contains numbers in the ‘Period 2’ and ‘Period 3’ columns that are more than half the number in the
‘Period 1’ column.
By quickly comparing the ‘Period 1’ and ‘Period 2’ columns, you can eliminate all but France, Belgium,
Denmark, Netherlands and Germany. By comparing ‘Period 1’ and ‘Period 3’ you can eliminate all but Belgium.
So the correct answer is A.
A 5° – 12°
B 7° – 9°
C 8° – 10°
D 8° – 9°
E 7° – 10°
Solution
The answer is D. The method here is to search for the acceptable highest and lowest temperatures for the
conditions to be met, realising that the middle value is irrelevant. As one reads 7°, the temperature cannot be
above 9° and, as another reads 10°, the temperature cannot be below 8°. This is given by D.
A This is obtained by subtracting 2 from the lowest and adding 2 to the highest.
E takes the range to be from the lowest reading to the highest reading.
Which one of the following graphs could show the actual change in the bank balance each month?
Solution
To solve this problem, you must first be clear about how the two types of graph represent the same information.
The main graph shows the balance at the end of each month; the graphs in the options show us the change in
the balance during each month. So, for example, the bar for February in the options represents the difference
between the bars for January and February in the main graph.
In the main graph, the balance goes down between the end of January and the end of February, so the bar for
February in the options should be negative. A comparison of the options shows that this is true only for options
A, C and D, so options B and E can be excluded. By comparing the values for each month in this way, you
should find that the correct option is D.
What is the largest number of seats that the third largest party can have?
A 22
B 118
C 119
D 120
E 121
Solution
Five parties share 400 seats. For the third largest party to have the maximum number of seats, the other parties
must have the minimum number, whilst still meeting the other conditions set out in the question. So the fourth
and fifth largest parties will have 21 and 20 seats respectively. This leaves 359 seats to be divided between the
three largest parties.
For the third largest party to have as many seats as possible, the other two must have only slightly more seats.
If we divide the remaining 359 seats as nearly as possible into thirds, we get: 1st = 120; 2nd = 120; 3rd = 119.
However, this violates the condition that no two parties have the same number of seats. To avoid this, one of
the seats of the third largest party must be transferred to the largest party.
This gives: 1st = 121; 2nd = 120; 3rd = 118; 4th = 21; 5th = 20. The answer is B.
A8
B 20
C 29
D 160
E 167
Solution
If the blue is the smallest, it is worth 1k. For the newspaper, then, the 33k is made up of 1k blue plus 32k
divided by 4 green, so each green is worth 8k.
We can now use this information to work out how many Ks one red note is worth: 135K + 3 x 8 (green coins) +
1k (1 blue) = 160k.
So, one red note is worth 160k. One green coin is worth 8k. On red note is worth 20 green coins.
It’s possible, of course, that the green is the smallest, but if this is so, it is worth 1k and the blue would be
worth 29k (33 – 4). The red note would then be worth 135 + 3 + 29 = 167k. This is not a multiple of 29,
however, and the question states that each higher denomination is a multiple of the lower denominations.
The answer is B.
What is the greatest amount of time that she could be late waking up?
A 1h 39min
B 1h 59min
C 2h 25min
D 2h 35 min
E 2h 59min
Solution
You are told that the time cannot be more than twelve hours later than seven o’clock in the morning, so you can know
that the first digit must be 0 (as it has the top element showing).
The visible elements of the other digits allow for the possibility that the time is the latest it can be before the first digit
changes to 1. That would be at 10:00 so this means that the time could be as late as 09:59.
Percentage distribution of
estimated salt intake (g/day)
Men Women
3 g or less 1 3
6 g or less 11 30
9 g or less 44 73
12 g or less 68 93
15 g or less 92 99
18 g or less 96 100
Assuming equal numbers of men and women in the population, what percentage of all people
consumed above the recommended level of 6g per day? (Give your answer to the nearest 1%)
A 20%
B 59%
C 70%
D 80%
E 89%
Solution
The only information given in the table that is relevant to this question is that 11% of the men and 30% of the women
consume no more than the recommended level of 6 g of salt per day.
If you assume, as you are told to, that there are equal numbers of men and women in the population, then, on average,
89 + 70 = 159 out of every 200 people consumed above the recommended level. This is 79.5%, or 80% to the nearest
1%.
Which of the following shapes could NOT be made from this table?
In this question you need to visualise the effect of rotating the pieces of the table around the hinges.
A can be achieved by rotating the right piece 45° clockwise and the left piece 45° anticlockwise.
B can be achieved by rotating the right piece 180° anticlockwise and the left piece 90° anticlockwise.
C can be achieved by rotating the right piece 180° anticlockwise and the left piece 180° clockwise.
E can be achieved by rotating the right piece 90° clockwise and the left piece 45° anticlockwise.
D would require either the right piece to be rotated 90° clockwise or the left piece to be rotated 90° anticlockwise
and the other piece to be detached from its hinge before it could be put in place.
My mother asked me to buy a Lottery ticket for her. She said “Choose any six numbers you
like, but make sure that each one is a multiple of 3, or contains a 3, or 6 or 9.”
A 15
B 16
C 22
D 25
E 31
Solution
There are a number of ways that this question can be approached, including the following:
The Nazarbayev University Foundation Year Program entrance test (‘NUFYP test’) is
owned by Nazarbayev University and Cambridge Assessment Admissions Testing;
the NUFYP test structure is owned by Nazarbayev University; the individual test items
are owned by Cambridge Assessment Admissions Testing and licensed to
Nazarbayev University in accordance with the terms and conditions of the licence of
use granted by Cambridge Assessment Admissions Testing. Any use of the NUFYP
test must be in accordance with the instructions of Nazarbayev University.